Which of these is the result of form having the illusion of the third dimension?

Answers

Answer 1

Depth is the result of form having the illusion of the third dimension.

Even while the human brain perceives many flat pictures, such as those in movies and photographs, as being two dimensional, nothing actually exists without all three dimensions (2D). This is due to the fact that everything physically existing is made of atoms, which are imperceptible to the human sight yet include all three spatial dimensions. Human eyes have the capacity to perceive "depth" or three dimensions. The three spatial dimensions of the world may be perceived by someone who possesses depth perception. Each human eye's visual brain first interprets the three spatial dimensions as 2D pictures. Contrarily, stereoscopic vision, which is unique to humans, alters the image seen by the two eyes.

Learn about three dimensions at

https://brainly.com/question/30211973

#SPJ4


Related Questions

What is the interquartile range for data set? 27,4,54,78,27,48,79,64,5,6,41,71

Answers

The Interquartile range for the given data set is 51.

The interquartile range for a given data set, the values of the first quartile (Q1) and the third quartile (Q3). The interquartile range is the difference between Q3 and Q1.

First, let's arrange the data set in ascending order:

4, 5, 6, 27, 27, 41, 48, 54, 64, 71, 78, 79

To find Q1, which represents the lower quartile, we need to locate the median of the lower half of the data set. Since the data set has 12 values, the lower half consists of the first 6 values:

4, 5, 6, 27, 27, 41

The median of this lower half is the average of the middle two values, which are 6 and 27:

Q1 = (6 + 27) / 2 = 33 / 2 = 16.5

To find Q3, the upper quartile, we need to locate the median of the upper half of the data set. Again, since the data set has 12 values, the upper half consists of the last 6 values:

48, 54, 64, 71, 78, 79

The median of this upper half is the average of the middle two values, which are 64 and 71:

Q3 = (64 + 71) / 2 = 135 / 2 = 67.5

Finally, we can calculate the interquartile range by subtracting Q1 from Q3:

Interquartile range = Q3 - Q1 = 67.5 - 16.5 = 51

Therefore, the interquartile range for the given data set is 51.

To know more about Interquartile .

https://brainly.com/question/4102829

#SPJ11

Write 0.75 as a fraction in its simplest form

Answers

Answer:

0.75 can be written as :-

[tex] \frac{75}{100} [/tex]

and in simplest form it is:-

[tex] \frac{3}{4} [/tex]

so the answer is 3/4

Cual es la distancia que recorrió luis en su bicicleta rodada 20p (2.54) después que las llantas dieran 50 vueltas completas



porfaaa

Answers

Luis traveled approximately 31,736.8 inches on his bicycle.

We have,

To find the distance that Luis traveled on his bicycle, we need to calculate the circumference of the tires and then multiply it by the number of complete turns.

Given:

Radius of the tires (r) = 20p (2.54) inches

Number of complete turns (n) = 50

The circumference of a circle can be calculated using the formula:

Circumference = 2πr

Substituting the given radius into the formula, we have:

Circumference = 2π * (20p) inches

Now we can calculate the distance traveled (d):

Distance = Circumference x Number of complete turns

Distance = 2π x (20p) x 50 inches

To simplify the calculation, we can approximate π as 3.14:

Distance ≈ 2 x 3.14 x (20 x 2.54) x 50 inches

Calculating this expression, we find:

Distance ≈ 31736.8 inches

Therefore,

Luis traveled approximately 31,736.8 inches on his bicycle.

Learn more about Circle here:

https://brainly.com/question/11833983

#SPJ1

The complete question.

What is the distance that Luis traveled on his bicycle rolled 20p (2.54) after the tires gave 50 complete turns

evaluate 5x - 2y + 4z when x=3 , y=2 and z=4 (a)5 (b) 16 (c) 27 (d) 20​

Answers

When x = 3, y = 2, and z = 4, the value of the expression 5x - 2y + 4z is 27.

The correct answer is (c) 27.

To evaluate the expression 5x - 2y + 4z when x = 3, y = 2, and z = 4, we substitute the given values into the expression and perform the   arithmetic calculations. Here's the step-by-step process:

Step 1: Replace x with 3, y with 2, and z with 4 in the expression:

5(3) - 2(2) + 4(4)Step 2:

Perform the multiplications first:

15 - 4 + 16

Step 3: Perform the additions and subtractions from left to right:

15 - 4 + 16 = 11 + 16 = 27.

For such more questions on  Expression:

https://brainly.com/question/25968875

#SPJ11

Can someone please answer and provide an explanation for these problems?

Answers

The center and radius for each equation are as follows:

40. Center: (3, 2), Radius: 8

41. Center: (-8, 4), Radius: 6

42. Center: (-4, 12), Radius: 2

43. Center: (4, -15), Radius: 3

What is the center and radius of the equations?

The standard equations of a circle is given as (x - h)² + (y - k)² = r²

Where the center are (h, k) and the radius of the circle is r.

40. (x - 3)² + (y - 2)² = 64

  Center: (3, 2)

  Radius: √64 = 8

41. (x + 8)² + (y - 4)² = 36

  Center: (-8, 4)

  Radius: √36 = 6

42. (x + 4)² + (y - 12)² = 4

  Center: (-4, 12)

  Radius: √4 = 2

43. (x - 4)² + (y + 15)² = 9

  Center: (4, -15)

  Radius: √9 = 3

Learn more on equation of circle here;

https://brainly.com/question/24810873

#SPJ1

The volume of a tree stump can be modeled by considering it as a right cylinder. Xavier measures its height as 2.1 ft and its circumference as 61 in. Find the volume of the stump in cubic inches. Round your answer to the nearest tenth if necessary.

Answers

The volume of the stump is 7451.9 cubic inches.

How to find the volume of the stump in cubic inches?

The volume of a cylinder can be calculated using formula below:

V = πr²h

where r is the radius and h is the height of the cylinder

We have circumference (C) = 61 in.

Let's find the radius (r) using the formula:

C = 2πr

61 = 2 * 22/7 * r

r = 9.70 in

h = 2.1 ft  = 2.1 * 12 = 25.2 in

Substituting into V = πr²h:

V = 22/7 * 9.70² * 25.2

V = 7451.9 cubic inches

Learn more about volume of cylinder on:

https://brainly.com/question/27535498

#SPJ1

En el laboratorio de análisis de minerales unas pequeñas gotas de ácido Nítrico (HNO3); cae sobre la piel de un analista y le produce una quemadura. ¿Cuántas moléculas de HNO3 provocaron la quemadura si las gotas presentan una masa de 0,49 g?

Answers

Entonces n is 0,00779 moles,

Los casi 5 sextillones (5 con 21 ceros) de moléculas de HNO3 provocaron la quemadura en la piel del analista.

Para responder a esta pregunta, necesitamos saber la masa molar de HNO3, que es 63,01 g/mol. Usando esta información, podemos calcular el número de moles de HNO3 en los 0,49 g de gotas usando la fórmula: n = m/M, donde n es el número de moles, m es la masa y M es la masa molar.

Como un mol contiene el número de moléculas de Avogadro (6,022 x 10^23), podemos calcular el número de moléculas de HNO3 en las gotas que causaron la quemadura: 0,00779 moles x 6,022 x 10^23 moléculas/mol = 4,69 x 10^21 moléculas .

Es importante recordar que los ácidos concentrados como el HNO3 pueden ser extremadamente peligrosos y pueden causar quemaduras graves, por lo que siempre se deben tomar las precauciones de seguridad adecuadas en un entorno de laboratorio.

To learn more about : molecules

https://brainly.com/question/29166394

#SPJ11

Find the area of the stained glass window shown if the diameter of the semi-circle is 61 inches. Use 3.14 for , round your answer to the nearest square inch, and enter the number only.

Answers

Answer:

  1460 square inches

Step-by-step explanation:

You want the area of a semicircle with diameter 61 inches.

Area

The area of a circle is given by ...

  A = (π/4)d²

The area of a semicircle is half that, so is ...

  A = (π/8)d²

  A = (3.14/8)(61 in)² ≈ 1460 in²

The area of the semicircular window is about 1460 square inches.

<95141404393>

Intro
Nautilus Clothing's stock has a 50% chance of producing a 15% return, a 20%
chance of producing a 21% return, and a 30% chance of producing a -13% return.
Part 1
What is the stock's expected return?

Answers

To calculate the stock's expected return, we need to multiply the probability of each return by its corresponding percentage return, and then sum up the results.

Let's calculate it step by step:

Return 1: 15% * 50% = 0.15 * 0.5 = 0.075 (or 7.5%)
Return 2: 21% * 20% = 0.21 * 0.2 = 0.042 (or 4.2%)
Return 3: -13% * 30% = -0.13 * 0.3 = -0.039 (or -3.9%)

Now, let's sum up the results:

Expected return = 0.075 + 0.042 - 0.039 = 0.078 (or 7.8%)

Therefore, the stock's expected return is 7.8%.

PLS HELP ME, WILL GIVE BRAINLIEST!!!!!!!!!!!!!

Answers

sorry i was going to show work but my water spill on the paper. so answer is 556.05cm^2

100 Points! Geometry question. Photo attached. Please show as much work as possible. Thank you!

Answers

Answer:

SRTZ is not a parallelogram.

Slope of SZ:

[tex]m = \frac{ - 2 - 1}{1 - ( - 2)} = - 1[/tex]

Slope of RT:

[tex]m = \frac{0 - 3}{2 - 1} = - 3[/tex]

Since the slopes of SZ and RT are not equal, SRTZ is not a parallelogram.

NO LINKS!! URGENT HELP PLEASE!!!

Solve ΔABC using the Law of Cosines

1. B= 36°, c = 19, a = 11

2. a = 21, b = 26, c = 17

Answers

Answer:

1)  A = 32.6°, C = 111.4°, b = 12.0

2) A = 53.6°, B = 85.7°, C = 40.7°

Step-by-step explanation:

Question 1

Given values of triangle ABC:

B= 36°c = 19a = 11

First, find the measure of side b using the Law of Cosines for finding sides.

[tex]\boxed{\begin{minipage}{6 cm}\underline{Law of Cosines (for finding sides)} \\\\$c^2=a^2+b^2-2ab \cos (C)$\\\\where:\\ \phantom{ww}$\bullet$ $a, b$ and $c$ are the sides.\\ \phantom{ww}$\bullet$ $C$ is the angle opposite side $c$. \\\end{minipage}}[/tex]

As the given angle is B, change C for B in the formula and swap b and c:

[tex]b^2=a^2+c^2-2ac\cos(B)[/tex]

Substitute the given values and solve for b:

[tex]\implies b^2=11^2+19^2-2(11)(19)\cos(36^{\circ})[/tex]

[tex]\implies b^2=482-418\cos(36^{\circ})[/tex]

[tex]\implies b=\sqrt{482-418\cos(36^{\circ})}[/tex]

[tex]\implies b=11.9929519...[/tex]

Now we have the measures of all three sides of the triangle, we can use the Law of Cosines for finding angles to find the measures of angles A and C.

[tex]\boxed{\begin{minipage}{7.6 cm}\underline{Law of Cosines (for finding angles)} \\\\$\cos(C)=\dfrac{a^2+b^2-c^2}{2ab}$\\\\\\where:\\ \phantom{ww}$\bullet$ $C$ is the angle. \\ \phantom{ww}$\bullet$ $a$ and $b$ are the sides adjacent the angle. \\ \phantom{ww}$\bullet$ $c$ is the side opposite the angle.\\\end{minipage}}[/tex]

To find the measure of angle A, swap a and c in the formula, and change C for A:

[tex]\implies \cos(A)=\dfrac{c^2+b^2-a^2}{2cb}[/tex]

[tex]\implies \cos(A)=\dfrac{19^2+(11.9929519...)^2-11^2}{2(19)(11.9929519...)}[/tex]

[tex]\implies \cos(A)=0.842229094...[/tex]

[tex]\implies A=\cos^{-1}(0.842229094...)[/tex]

[tex]\implies A=32.6237394...^{\circ}[/tex]

To find the measure of angle C, substitute the values of a, b and c into the formula:

[tex]\implies \cos(C)=\dfrac{a^2+b^2-c^2}{2ab}[/tex]

[tex]\implies \cos(C)=\dfrac{11^2+(11.9929519...)^2-19^2}{2(11)(11.9929519...)}[/tex]

[tex]\implies \cos(C)=-0.364490987...[/tex]

[tex]\implies C=\cos^{-1}(-0.364490987...)[/tex]

[tex]\implies C=111.376260...^{\circ}[/tex]

Therefore, the remaining side and angles for triangle ABC are:

b = 12.0A = 32.6°C = 111.4°

[tex]\hrulefill[/tex]

Question 2

To solve for the remaining angles of the triangle ABC given its side lengths, use the Law of Cosines for finding angles.

[tex]\boxed{\begin{minipage}{7.6 cm}\underline{Law of Cosines (for finding angles)} \\\\$\cos(C)=\dfrac{a^2+b^2-c^2}{2ab}$\\\\\\where:\\ \phantom{ww}$\bullet$ $C$ is the angle. \\ \phantom{ww}$\bullet$ $a$ and $b$ are the sides adjacent the angle. \\ \phantom{ww}$\bullet$ $c$ is the side opposite the angle.\\\end{minipage}}[/tex]

Given sides of triangle ABC:

a = 21b = 26c = 17

Substitute the values of a, b and c into the Law of Cosines formula and solve for angle C:

[tex]\implies \cos(C)=\dfrac{a^2+b^2-c^2}{2ab}[/tex]

[tex]\implies \cos(C)=\dfrac{21^2+26^2-17^2}{2(21)(26)}[/tex]

[tex]\implies \cos(C)=\dfrac{828}{1092}[/tex]

[tex]\implies C=\cos^{-1}\left(\dfrac{828}{1092}\right)[/tex]

[tex]\implies C=40.690560...^{\circ}[/tex]

To find the measure of angle B, swap b and c in the formula, and change C for B:

[tex]\implies \cos(B)=\dfrac{a^2+c^2-b^2}{2ac}[/tex]

[tex]\implies \cos(B)=\dfrac{21^2+17^2-26^2}{2(21)(17)}[/tex]

[tex]\implies \cos(B)=\dfrac{54}{714}[/tex]

[tex]\implies B=\cos^{-1}\left(\dfrac{54}{714}\right)[/tex]

[tex]\implies B=85.6625640...^{\circ}[/tex]

To find the measure of angle A, swap a and c in the formula, and change C for A:

[tex]\implies \cos(A)=\dfrac{c^2+b^2-a^2}{2cb}[/tex]

[tex]\implies \cos(A)=\dfrac{17^2+26^2-21^2}{2(17)(26)}[/tex]

[tex]\implies \cos(A)=\dfrac{524}{884}[/tex]

[tex]\implies A=\cos^{-1}\left(\dfrac{524}{884}\right)[/tex]

[tex]\implies A=53.6468753...^{\circ}[/tex]

Therefore, the measures of the angles of triangle ABC with sides a = 21, b = 26 and c = 17 are:

A = 53.6°B = 85.7°C = 40.7°

Write and solve an equation to find the value of x.

Answers

The equation to solve this is:

20 (20+30) = x^2
1000 = x^2
sqr root of 1000 = x

x = 31.62277660168379
x ≈ 31.62

Find the value for the side marked below. Round your answer to the nearest tenth 210 37 degrees

Answers

Using the cosine ratio, the value of the marked side in the image given below is approximately: y = 167.7.

How to Find the Value of the Marked Side Using the Cosine Ratio?

The cosine ratio is defined as the ratio of the length of the hypotenuse of the right triangle over the length of the side that is adjacent to the reference angle. It is given as:

cos ∅ = length of hypotenuse/length of adjacent side.

From the image attached below, we have the following:

Reference angle (∅) = 37°

length of hypotenuse = 210

length of adjacent side = y

Plug in the values:

cos 37 = y/210

210 * cos 37 = y

y = 167.7

Learn more about the cosine ratio on:

https://brainly.com/question/15793827

#SPJ1

Help I need the answer to this

Answers

The graph of the logarithmic function is attached below with a vertical asymptote at x = -8 and two integer coordinates at (2, -7) and (1, -10).

What is graph of a logarithmic function?

The basic logarithmic function is of the form f(x) = logax (r) y = logax, where a > 0. It is the inverse of the exponential function ay = x. Log functions include natural logarithm (ln) or common logarithm (log).

To plot the graph of the given function, we simply need to use a graphing calculator.

The given function is;

f(x) = -3log₃(x + 8) - 4

To find the asymptotes of the graph;

x + 8 > 0

x > -8

The vertical asymptotes is at x = -8

The two points with integer coordinates are (2, -7) and (1, -10)

Learn more on graph of logarithmic function here;

https://brainly.com/question/30194309

#SPJ1

what is 27% in a equivalent form using the two other forms of notian: fraction,decimal,or percent

Answers

You can write 27% as a fraction like this: [tex]\frac{27}{100}[/tex] . (27/100).

Or as a decimal 0.27.

helpppppppppp meeeeee please ​

Answers

The missing sides of the triangle are given as follows:

[tex]SX = 4\sqrt{3}[/tex]RS = 12.

What are the trigonometric ratios?

The three trigonometric ratios are the sine, the cosine and the tangent of an angle, and they are obtained according to the formulas presented as follows:

Sine = length of opposite side to the angle/length of hypotenuse of the triangle.Cosine = length of adjacent side to the angle/length of hypotenuse of the triangle.Tangent = length of opposite side to the angle/length of adjacent side to the angle = sine/cosine.

For the angle of 30º, we have that:

6 is the opposite side.RS is the hypotenuse.

Hence the hypotenuse RS is obtained as follows:

sin(30º) = 6/RS

1/2 = 6/RS

RS = 6 x 2

RS = 12.

Applying the cosine, we have that:

cos(30º) = 6/SX

[tex]\frac{\sqrt{3}}{2} = \frac{6}{SX}[/tex]

[tex]SX = \frac{12}{\sqrt{3}}[/tex]

[tex]SX = 4\sqrt{3}[/tex]

A similar problem, also about trigonometric ratios, is given at brainly.com/question/24349828

#SPJ1

100 Points! Geometry question. Photo attached. Determine whether the pair of triangles is similar. If so, write a similarity statement. If not, what would be sufficient to prove the triangles similar? Explain your reasoning. Please show as much work as possible. Thank you!

Answers

Answer:

∆TSU ~ ∆PJM by SAS since 10/14 = 5/7 and 15/21 = 5/7, and angle S is congruent to angle J.

What is the value of x? Show all your work.


Please help. 100 points.

Answers

Answer:

12 cm

Step-by-step explanation:

By hypotenuse theorem,

x² + 35² = 37²

x² + 35*35 = 37*37

x² + 1225 = 1369

x² = 1369 - 1225

x² = 144

x² = 12*12

x² = 12²

x = 12 cm

Please help me, first time I got it wrong

Answers

The conditional value probability is solved and P ( F | E ) = 6/17

Given data ,

P ( E ) = 0.85

P ( F ) = 0.4

P ( E ∩ F ) = 0.3

Now , the formula for conditional probability to calculate P(F|E):

P(F|E) = P(E ∩ F) / P(E)

We are given that P(E) = 0.85 and P(E ∩ F) = 0.3, so we can substitute those values in:

P(F|E) = 0.3 / 0.85

Simplifying this fraction, we get:

P(F|E) = 6/17

Hence , the probability of F given that E has occurred is 6/17 or approximately 0.35.

To learn more about probability click :

https://brainly.com/question/17089724

#SPJ1

Sobre una embarcación de 160 kg que está en reposo con su proa apuntando a la orilla, comienza a caminar una persona de 70 kg desde la proa hacia la popa, a 0.80 m/s respecto a la embarcación. ¿Cuáles son las velocidades de la embarcación y de la persona respecto a la orilla? Desprecia la resistencia del agua al movimiento.

Answers

The velocities of the boat and the person relative to the shore are 1.337 m/s and 2.896 m/s, respectively.

How to calculate the velocity

We can use the conservation of momentum equation:

(mboat + mperson) * vboat = mperson * vperson

(160 kg + 70 kg) * vboat = 70 kg * (0.80 m/s + vperson)

230 kg * vboat = 56 kg * (0.80 m/s + vperson)

vboat = (56/230) * (0.80 m/s + vperson)

vperson = 0.80 m/s + vboat

vperson = 0.80 m/s + (56/230) * (0.80 m/s + vperson)

(174/115) * vperson = (504/115) * m/s

Dividing both sides by (174/115), we get:

vperson = 2.896 m/s

vboat = (56/230) * (0.80 m/s + vperson)

Substituting the value of vperson, we get:

vboat = 1.337 m/s

Learn more about velocity on

https://brainly.com/question/25749514

#SPJ1

On a 160-kg boat that is at rest with its bow pointed to the shore, a 70-kg person begins to walk from the bow to the stern at 0.80 m/s relative to the boat. What are the velocities of the boat and the person relative to the shore? Neglect the resistance of water to motion.

The table shows ordered pairs of the function y=8-2x. What is the value of y when x=8?
0-20
X
-3
-1
-
1
4
8
10
Mark this and return
y
14
10
6
0
?
-12
Save and Exit
Next
Submit

Answers

Answer:

The Value of Y is -8

Step-by-step explanation:

I believe it is -8 because giving the function, y = 8 - 2x, given that x = 8, we can replace the value into a function. y = 8 - 2(8) = 8 - 16 = -8.

So therefore, the value of y is -8. Hopes this helps :p

Find the volume of the pyramid above
Find the surface are of the pyramid above pls help

Answers

The volume of the pyramid is 18069333.33 units³ and the surface area is 391600 units ²

What is a pyramid?

A pyramid is a three-dimensional figure. It has a flat polygon base. All the other faces are triangles and are called lateral faces.

Surface area of a pyramid is expressed as;

area of 4 lateral face + area of base

area of base = 440²

= 193600 units²

area of a lateral = 1/2 bh

= 1/2 × 440 × 356

= 78320

For four surfaces = 4 × 78320 = 313280

Total surface area = 313280+78320

= 391600 units ²

Volume of a pyramid is expressed as;

V = 1/3base area × height

V = 1/3 × 440² × 280

V = 18069333.33 units³

learn more about pyramid from

https://brainly.com/question/218706

#SPJ1

Xavior took a total of 124 quarters and dimes to trade in for cash at the bank. He got exactly $25 back. How many quarters did he have?

A.
40

B.
62

C.
84

D.
100

Answers

C. 84 he had 84 quarters

Which point is located at 2,1

Answers

Answer: Point P

Step-by-step explanation: over 2 up 1 (2,1)

Answer: M

Step-by-step explanation:

Gauri Spends 0.75 of her salary every month if she earns rs 12000 per month in how many months will she save rupees 39000

Answers

Answer:

13 months

--------------------

If Gauri spends 0.75 of her salary every month, that means she saves 0.25 of her salary:

0.25 * 12000 = 3000

Divide the total 39000 by her monthly savings:

39000 / 3000 = 13

So it will take Gauri 13 months to save rupees 39000.

Answer and why!!!!!!

Answers

The expression (yx + y + px + p) / (5x² + 10x +5) * (10x + 10) / (y² + yp) is simplified to obtain

2/y

How to simplify the expression

The given expression is

(yx + y + px + p) / (5x² + 10x +5) * (10x + 10) / (y² + yp)

The expression is simplified individually, using different each equation in the expression

yx + y + px + p

= y(x + 1) + p(x + 1)

= (y + p)(x + 1)

5x² + 10x +5

= 5(x² + 2x + 1)

= 5(x² + 1)

(10x + 10)

= 10(x + 1)

(y² + yp)

= y(y + p)

bringing the equations together and simplifying further

(y + p)(x + 1) / 5(x² + 1) * 10(x + 1) / y(y + p)

= 10(x + 1)(x + 1) / 5y(x² + 1)

= 10(x² + 1) / 5y(x² + 1)

= 2/y

Learn more about simplifying at

https://brainly.com/question/28780542

#SPJ1

Determine if the given side lengths could be used to form a unique triangle, many different triangles, or no triangles.

3.4 cm, 3.1 cm, 6.6 cm

Answers

We can see here that the given side lengths cannot  be used to form a unique triangle because the shorter sides do not add up to the longer side.

What is a triangle?

The fundamental geometric shape of a triangle has three sides and three angles. It is a triangular polygon with three edges.

We can see here that in order to determine if the given side lengths:

3.4 cm, 3.1 cm, 6.6 cm

can form a unique triangle, many different triangles, or no triangle, we need to check if they satisfy the triangle inequality theorem.

According to the Triangle Inequality Theorem, the sum of the lengths of any two sides of a triangle must be greater than the length of the remaining side.

Let's check the conditions:

3.4 cm + 3.1 cm = 6.5 cm

6.5 cm > 6.6 cm (Not satisfied)

3.4 cm + 6.6 cm = 10 cm

10 cm > 3.1 cm (Satisfied)

3.1 cm + 6.6 cm = 9.7 cm

9.7 cm > 3.4 cm (Satisfied)

he specified side lengths cannot be used to create a triangle because the total of the two shorter sides' lengths (3.4 cm and 3.1 cm) is not larger than the length of the longest side (6.6 cm).

Learn more about triangle on https://brainly.com/question/17335144

#SPJ1

What is 1/3 of 343???????????

Answers

The number that is one-third of 343 is 49.

1/3 of 343 can be found by multiplying 343 by 1/3:

1/3 x 343 = (1/3) x (343) = 343/3

To simplify the result, we can divide both the numerator and denominator by the greatest common factor (GCF) of 343 and 3, which is 7:

343/3 = (343 ÷ 7) / (3 ÷ 7) = 49/1

Therefore, 1/3 of 343 is 49.

To learn more about the greatest common factor;

https://brainly.com/question/11221202

#SPJ1

please help. i’m struggling on part C

Answers

a) The area formula for the rectangle is equal to A = r² · sin 2θ.

b) By derivative tests, the maximum possible area of the rectangle is 16 square centimeters.

c) The dimensions of the rectangle are: Width: 5.66 cm, Height: 2.83 cm

How to find the maximum possible area of a rectangle inscribed in a semicircle

In this problem we must determine the maximum possible area of a rectangle inscribed in a semicircle by means of first and second derivative tests. First, derive the area formula of the rectangle:

A = w · h

A = (2 · r · cos θ) · (r · sin θ)

A = 2 · r² · sin θ · cos θ

A = r² · sin 2θ

Where:

w - Width, in centimeters.h - Height, in centimeters.A - Area, in square centimeters.r - Radius, in centiemters. θ - Angle, in degrees.

Second, perform first derivative test: (r - Constant)

A = 2 · r² · cos 2θ

2 · r² · cos 2θ = 0

cos 2θ = 0

θ = 45°

Third, perform second derivative test: (θ = 45°)

A'' = - 4 · r² · sin 2θ

A'' = - 4 · r² (MAXIMUM)

Fourth, determine the maximum possible area of the rectangle:

A = 4² · sin 90°

A = 16 cm²

Fifth, determine the width and the height of the rectangle: (r = 4, θ = 45°)

w = 2 · r · cos θ

w = 2 · 4 · cos 45°

w = 8 · √2 / 2

w = 4√2 cm

w = 5.66 cm

h = r · sin θ

h = 4 · sin 45°

h = 4 · √2 / 2

h = 2√2 cm

h = 2.83 cm

To learn more on derivative tests: https://brainly.com/question/30404403

#SPJ1

Other Questions
A sample of n = 16 scores produces a t statistic of t = 2.00. If the sample is used to measure effect size with r2, what value will be obtained for r2a. r2 = 2/20 c. r2 = 2/19b. r2 = 4/20 Andy's Accessories produces backpacks. The costs and prices for the backpacks follow (Assume the same unit costs in all years): Selling price $23.00 per backpack Variable costs: Production $11.00 per backpack $2.00 per backpack Selling Fixed Costs: Production $900,000 per year $540,000 per year Selling and administrative Andy's Accessories produced 250,000 backpacks for the year and sold 200,000. There was no beginning inventory, and costs throughout the year were stable. How much is the cost of ending inventory under full costing? $650,000 O $550,000 O $730,000 $938,000 cross-culturally and historically, marrrigaes based oin free choice and romancitc love are An antique skateboard has an area of 208 in. . The short sides of the rectangular port are each 8 inches long. Complete the model An isomer of C3H7O undergoes one step oxidation reaction. Answer the following questions due to this reaction.a) Write a full symbol equation for this reaction.b) Name the proper reagent and catalyst for this reaction.c) Why do you think there is no need to remove the product from the reaction vessel? compare the temperature change as pure liquid is converted to a solid as its freezing point with the temperature change as a solution is converted to a solid at its freezing? the following is a valid probability distribution. what is the p(x = 0)? x 0 1 2 3 4 5 p(x) 0.14 0.24 0.12 0.07 0.34 1. If the height of a parallelogram is 27.3 m and the base is 6.3m, what is the area of the parallelogram?2. If the base of a rectangle is 27.4 mm and the area is 420.92 mm2. What is the height of the rectangle?3. What is the height of a rectangle with a perimeter of 133.2 mm and a base length of 30.6 mm?4. If the base of a parallelogram is 61 m and the area is 645 m2, what is the height of the parallelogram5. What is the perimeter of a parallelogram with a base of 32 cm, a side length of 34 cm, and a height of 19 cm?6. What is the area of a parallelogram with a base of 40 mm, a side length of 15 mm, and height of 18 mm?7. If the height of a rectangle is 8.8 m and the base is 7.9 m, what is the perimeter of the rectangle8. If the base of a rectangle is 23cm and the area is 354 cm2, what is the height of the rectangle?9. What is the area of a square with a perimeter of 44 mm?10. What is the area of a square with a perimeter of 55 cm?Introduction to Perimeter & Area of Polygons The tenderness in pastry is achieved by all of the following:a. Blend soft fat into flour before adding any liquidb. Instead of water use an ingredient that's part fat.c. Add acid to dough regional and interregional trade only began to expand with the arrival of the europeans.T/F calculate delta g for an electrochemical cell reaction that occurs under basic aques condittitons based on the following two half-reactions for which the standard reduction potentials are given. Use the smallest whole-number coefficients possible when balancing the overall reaction. Cd(OH)2 + 2e- ---> Cd + 2OH- -0.824VNiO(OH) + H2O + e- ---> Ni(OH)2 + OH- +1.32V Increasing the displacement of a vibrating particle in a mechanical wave from the equilibrium position will increase: Consider the reaction C($) + CO,C) = 2008). At 1273 K, the Kp value is 167.5. What is the Peo at equilibrium if the Pro, is 0.25 atm at this temperature? O a. 9.2 atm O b.3.2 atm c. 13 atm d, 0.130 atm 0.6.5 atm 27 T/F. schizophrenia is associated with enlarged lateral ventricles, reflecting the shrinkage of brain tissue. (a) if cos 2 ( 29 ) sin 2 ( 29 ) = cos ( a ) , then the on-axis magnetic field strength 12 cm from a small bar magnet is 600 t. What is the bar magnet's magnetic dipole moment? you want to mount a number of file systems each time the system is brought up. which configuration file should hold the configuration information for the file systems to be mounted? The compensation point of fern plants which grow on the forest floor happens at 10. 00a. M. In your opinion ,at what time does a ficus plants which grows higher in the same forest achieve it's compensation point? A bank offers two different types of savings account which pay interest as shown below. Hannah wants to invest 3200 in one of these accounts for 13 years. a) Which account will pay Hannah more interest after 13 years? b) How much more interest will that account pay? Give your answer in pounds () to the nearest 1p. Account 1 Simple interest at a rate of 5% per year Account 2 Compound interest at a rate of 4% per year according to the module, major personality traits show heritabilities of ___.